Đến nội dung

minhtuyb nội dung

Có 497 mục bởi minhtuyb (Tìm giới hạn từ 29-04-2020)



Sắp theo                Sắp xếp  

#302813 ..Gọi BC=a, AB=c, AC=b, DH=x, DK=z, DI=y..Cm $\frac{a}{x}=\fra...

Đã gửi bởi minhtuyb on 07-03-2012 - 22:05 trong Hình học

http://diendantoanho...showtopic=68132
:D



#302815 Giải phương trình nghiệm nguyên ${x^3} + {y^3} = 1985$

Đã gửi bởi minhtuyb on 07-03-2012 - 22:12 trong Phương trình, hệ phương trình và bất phương trình

Ta có:
$1985=x^3+y^3=(x+y)(x^2-xy+y^2)$
Mà 1985=5.397
do đó:

Hình đã gửi

Từ đó tìm được x, y
___________________________________________________________
Quan trọng là cái đường lối

Còn 2 trường hợp :D:
$\left\{\begin{matrix}x+y=1985\\ x^2-xy+y^2=1\end{matrix}\right.$
$\left\{\begin{matrix}x+y=1\\ x^2-xy+y^2=1985\end{matrix}\right.$
Ai có cách nào ngắn hơn ko nhỉ :D



#302862 Topic bất đẳng thức THCS (2)

Đã gửi bởi minhtuyb on 08-03-2012 - 11:36 trong Bất đẳng thức và cực trị

Bài 292: Cho các số a,b,c thực dương. Chứng minh rằng
$\frac{a^3}{b^2}+\frac{b^3}{c^2}+\frac{c^3}{a^2}\geq \frac{a^2}{b}+\frac{b^2}{c}+\frac{c^2}{a}$

Thêm một cách nữa :D
Áp dụng BĐT Cauchy cho 19 số, ta có:
$14\frac{a^3}{b^2}+3\frac{b^3}{c^2}+2\frac{c^3}{a^2}\geq 19\sqrt[19]{\frac{a^{3.14}.b^{3.3}.c^{3.2}}{b^{2.14}.c^{2.3}.a^{2.2}}}=19\frac{a^2}{b}$
Xây dựng các BĐT tương tự rồi cộng vào ta có ĐPCM
Dấu bằng xảy ra khi $a=b=c$



#302885 Những bài toán chưa có lời giải Part 1

Đã gửi bởi minhtuyb on 08-03-2012 - 13:19 trong Bất đẳng thức và cực trị

24. (nguyen_ct -13-11-2008)
Cho $a,b,c$ và $x,y,z$ là các số dương

Tìm GTNN của Hình đã gửi biết $x+y+z=3$
25.
(nguyen_ct -13-11-2008)
Cho $a,b,c$ là các số dương
CM Hình đã gửi

25. Với $a=b=c=0,5$, BĐT sai
Chắc là ntnày: $\sum \frac{a^2b}{c}\geq \sum a^2$
24. Đặt $P=\sum \frac{ax}{b+c}\geq P+\sum x=\sum (\frac{ax}{b+c}+x)$
$\Rightarrow P+3=\sum \frac{(a+b+c)x}{b+c}=(a+b+c)\sum \frac{x}{b+c}$



#302895 tìm giá trị lớn nhất của $\sqrt{x+1}-\sqrt{x-8}$

Đã gửi bởi minhtuyb on 08-03-2012 - 14:51 trong Đại số

http://diendan.hocma...ad.php?t=184813
http://vn.answers.ya...29105656AAWtTaa
Tham khảo thêm nhé bạn :D



#302923 Topic bất đẳng thức THCS (2)

Đã gửi bởi minhtuyb on 08-03-2012 - 16:44 trong Bất đẳng thức và cực trị

Bài 294: Cho a,b,c thực dương chứng minh rằng
$\sqrt{2a(a+b)^3}+b\sqrt{2(a^2+b^2)}\le3(a^2+b^2)$


Sai đừng ném gạch nhé :lol:
$\sqrt{2a(a+b)^3}+b\sqrt{2(a^2+b^2)}=\sqrt{(2a^2+2ab)(a^2+2ab+b^2)}+\sqrt{2b^2(a^2+b^2)}$
$\leq ^{AM-GM} \frac{2a^2+2ab+a^2+2ab+b^2}{2}+\frac{2b^2+a^2+b^2}{2}=2(a^2+b^2)+2ab\leq 2(a^2+b^2)+a^2+b^2=3(a^2+b^2)<q.e.d>$
Dấu bằng xảy ra khi $a=b$
P/s: c đâu anh >:)

Gõ nhầm thôi mà :-j



#302965 Topic bất đẳng thức THCS (2)

Đã gửi bởi minhtuyb on 08-03-2012 - 20:32 trong Bất đẳng thức và cực trị

Bài 295: Cho x,y,z thực thỏa mãn $x^2+y^2+z^2=1-\frac {9}{16} xy$
Tìm GTLN của biểu thức $P=xy+xz+yz$
Bạn nào post mấy cái đề lên cho mọi người cùng làm :P

Oạch bài này dùng "Cân bằng hệ số à", sao đáp án của em lẻ quá :wacko:
$x^2+y^2+z^2=1-\frac {9}{16} xy\Leftrightarrow x^2+y^2+z^2+\frac {9}{16} xy=1(1)$
Dễ thấy vai trò của $x,y$ như nhau nên xác định được điểm rơi ở $x=y=tz$ (t là tham số dương). Vậy cần xác định tham số $a,t$ sao cho (1) tương đương với phương trình sau:
$a(x-y)^2+(x-tz)^2+(y-tz)^2\geq 0(2)$
BIến đổi:
$(2)\Leftrightarrow ax^2+ay^2-2axy+x^2+t^2z^2-2txz+y^2+t^2z^2-2tyz\geq 0$
$\Leftrightarrow (a+1)x^2+(a+1)y^2+2t^2z^2\geq 2axy+2tyz+2txz$
$\Leftrightarrow (a+1)x^2+(a+1)y^2+2t^2z^2+\frac{9}{16}(a+1)xy\geq (\frac{41}{16}a+\frac{9}{16})xy+2tyz+2txz$
Từ đó ta có cách chọn tham số $a,t$ như sau:
$\left\{\begin{matrix}a+1=2t^2\\ \frac{41}{16}a+\frac{9}{16}=2t\end{matrix}\right.$
Tìm được tham số rồi thay ngược lên là ra
Nhưng số xấu quá :ukliam2:
P/s: Ai có cách nào khác không :D



#303075 1 cho pt $ x^2-4x+m-1=0$ tìm m để phương trình có nghiệm trái dấu

Đã gửi bởi minhtuyb on 09-03-2012 - 12:58 trong Đại số

a. PT có nghiệm trái dấu thì chỉ cần $ac<0$ là được, vì $ac<0$ thì hiển nhiên $\Delta =b^2-4ac$ hiển nhiên dương:
$ac=m-1<0\Leftrightarrow m<1$
b. $\left\{\begin{matrix}\Delta '=5-m>0\\x_1.x_2= m-1>0\\ x_1+x_2=4>0\end{matrix}\right.\Leftrightarrow 1<m<5$



#303091 Để hiểu hơn về phương pháp quy nạp toán học

Đã gửi bởi minhtuyb on 09-03-2012 - 16:20 trong Chuyên đề toán THPT

Hì hóa ra lớp 11 mới chính thức học cái quy nạp toán học, giờ em mới biết thế :icon6:
Bài 3: Tính tổng sau theo $n$ (Dự đoán vs c/m):
$S=1^4+2^4+3^4+...+n^4$
Bài 4: Chứng minh rằng nếu n là một hợp số lớn hơn 4 thì ta có tích P = 1.2.3...(n - 1) chia hết cho n.
Bài 5:Chứ ng minh rằng với mọi số nguyên đồng (tiền Việt Nam) lớn hơn 6 có thể đổi ra tiền lẻ không dư bằng những đồng tiền gồm những tờ 2 đồng và 5 đồng

Chắc chất lượng bài không đảm bảo :lol:



#303281 Trận 4 - "MSS04 nguyenta98ka" VS ALL

Đã gửi bởi minhtuyb on 10-03-2012 - 11:40 trong Thi giải toán Marathon cấp THCS 2012

Đề hiền nhất từ đầu mùa giải đến giờ :wub: :wub: :wub:
Hình đã gửi
Bài làm của minhtuyb:
-Nối $AM;IK$. Tứ giác $AIMK$ là hình chữ nhật do có $\widehat{BAC}=\widehat{AKM}=\widehat{AIM}=90^o\Rightarrow AM=IK$
Mặt khác, áp dụng đ/lý Py-ta-go vào $\Delta MIK$ có $MI^2+MK^2=AM^2=IK^2(1)$
Vậy $MI^2+Mk^2+MH^2$ đạt min khi và chỉ khi $AM^2+MH^2$ đạt min.
-Hạ đường cao $AH'$ của $\Delta ABC\Rightarrow AH$ có độ dài không đổi, từ M hạ $MM'\perp AH'\Rightarrow$ tứ giác $MM'H'H$ là hình chữ nhật (có 3 góc vuông) $\Rightarrow MH=M'H'\Rightarrow MH^2=M'H'^2(2)$
-Theo quan hệ giữa đường xiên và hình chiếu, có $AM\geq AM'\Rightarrow AM^2\geq AM'^2(3)$
-Từ (2) và (3) suy ra:
$AM^2+MH^2\geq AM'^2+M'H'^2$
*Áp dụng BĐT $x^2+y^2\geq \frac{(x+y)^2}{2} \Leftrightarrow (x-y)^2\geq 0$ (Luôn đúng), có:
$AM^2+MH^2\geq AM'^2+M'H'^2\geq \frac{(AM'+M'H')^2}{2}=\frac{AH'^2}{2}=const(4)$
-Từ (1) và (4)$\Rightarrow MI^2+MK^2+MH^2\geq \frac{AH'^2}{2}$
Dấu bằng xảy ra khi $M\equiv M'\in AH'$ là đường cao hạ từ $A$ xuống cạnh $BC$
Vậy $min(MI^2+MK^2+MH^2)=\frac{AH'^2}{2}$ khi $M\in AH'$ là đường cao hạ từ A xuống cạnh $BC$ của $\Delta ABC(M\not\equiv A;H)$

Mong là sẽ về đầu tiên :icon6:

Bố sung của minhtuyb:
Cho bổ sung phần dấu bằng :P:
Dấu bằng xảy ra khi $M\equiv M'\in AH$ và $AM'=M'H'\Leftrightarrow M\equiv M'$ là trung điểm của $AH'$
Vậy $min(MI^2+MH^2+MK^2)=\frac{AH'^2}{2}$ khi $M$ là trung điểm $AH'$, với $AH'$ là đường cao hạ từ đỉnh A xuống cạnh BC của $\Delta ABC$

Hút chết >:) >:) >:)

Kết quả:
D-B=14.3h
E=10
F=0
S=63.7



#303328 Topic bất đẳng thức THCS (2)

Đã gửi bởi minhtuyb on 10-03-2012 - 13:22 trong Bất đẳng thức và cực trị

Bài 300: Cho x,y,z dương tích bằng 1. CMR:
$9+ \frac{1}{x^3}+\frac{1}{y^3}+\frac{1}{z^3}\geq (x+y+z)(xy+yz+zx)+ \frac{(xy+yz+zx)^2}{x+y+z}$
<Của anh bboy :icon10: >



#303380 Tìm tất cả các số nguyên dương $a,b,c$ $a^{3}+b^{3}+c^{3}=2001...

Đã gửi bởi minhtuyb on 10-03-2012 - 19:48 trong Số học

Đúng ra đề này là $a^3+b^3+c^3=2011$ chứ (thấy bài này trong cái Chuyên đề của Tạ Văn Đức)
Cũng xét mod 9 như trên :D



#303393 $$\text{Tính tổng:}\,\, a^3+b^3+c^3$$

Đã gửi bởi minhtuyb on 10-03-2012 - 20:13 trong Đại số

Bài 1: Là sao, đã biết cái gì rồi :ph34r:
Bài 2:
C1:Quy nạp <Lười lắm sẽ post nếu có t/gian :icon6: >

C2:Nhanh hơn:
$S=1.4+2.5+...+n(n+3)=(1^2+3.1)+(2^2+3.2)+(3^2+3.3)+...+(n^2+3n)$
$=(1^2+2^2+3^2+...+n^2)+3(1+2+3+...+n)$
$=\frac{n(n+1)(2n+1)}{6}+\frac{n(n+1)}{3}=...=\frac{n(n+1)(n+5)}{3}$



#303469 Chứng minh bốn đường tròn phủ kín tứ giác

Đã gửi bởi minhtuyb on 10-03-2012 - 22:44 trong Hình học

-Gọi M là một điểm bất kì của tứ giác lồi ABCD. Nối M với các đỉnh. Xét 2 trường hợp:
TH1:Nếu M nằm trên cạnh tứ giác thì dĩ nhiên ta thấy M nằm trong đường tròn với đường kính là cạnh đó
TH2:
-Nếu M nằm trong tứ giác lồi ABCD . Khi đó ta có
$\widehat{AMB}+\widehat{BMC}+\widehat{CMD}+\widehat{DMA}=360^o$
-Theo nguyên lí cực hạn tồn tại góc lớn nhất trong 4 góc trên , giả sử đó là $\widehat{AMB}\Rightarrow \widehat{AMB}\geq 90^o$
$\Rightarrow$ M nằm trong(hoặc trên) đường tròn đường kính AB. Vậy dĩ nhiên M bị phủ bởi đường tròn này.
Như thế do M là một điểm tùy ý trong tứ giác lồi ABCD , ta suy ra bốn hình tròn trên phủ kín tứ giác lồi đã cho. Vậy ta có dpcm
Từ đây: http://diendan.hocma...t=186471&page=6



#303594 Topic bất đẳng thức THCS (2)

Đã gửi bởi minhtuyb on 11-03-2012 - 17:22 trong Bất đẳng thức và cực trị

Oạch anh Kiên xóa cả lời giải bài 299 của em rồi :(

Bài 299: Cho a,b là 2 số thực dương thay đổi thỏa mãn ab=1.Chứng minh rằng
$\frac{a^3}{1+b^2}+\frac{b^3}{1+a^2}\geq 1$

C1:$\frac{a^3}{1+b^2}+\frac{b^3}{1+a^2}=\frac{a^4}{a+ab^2}+\frac{b^4}{b+a^2b}=\frac{a^4}{a+b}+\frac{b^4}{a+b}\geq \frac{(a^2+b^2)^2}{2(a+b)}$
$\geq \frac{\frac{(a+b)^4}{4}}{2(a+b)}=\frac{(a+b)^3}{8}\geq \frac{(2\sqrt{ab})^3}{8}=1<Q.E.D>$
Dấu bằng xảy ra khi $a=b=1$
C2: Cauchy ngược.
$\frac{a^3}{1+b^2}+\frac{b^3}{1+a^2}=a^3-\frac{a^3b^2}{b^2+1}+b^3-\frac{a^2b^3}{a^2+1}\geq a^3-\frac{a^2b}{2b}+b^3-\frac{ab^2}{2a}$
$=(a+b)(a^2-ab+b^2)-(\frac{a^2}{2}+\frac{b^2}{2})=(a+b)(a^2+b^2-1)-\frac{1}{2}(a^2+b^2-1)-\frac{1}{2}=(a+b-\frac{1}{2})(a^2+b^2-1)-\frac{1}{2}\geq (2\sqrt{ab}-\frac{1}{2})(2ab-1)-\frac{1}{2}=1<Q.E.D>$
Cách cauchy ngược trông "trâu bò" hơn :P



#303607 Cho x,y>0 , x+y= 100,$x\geq 60$ tìm max xy

Đã gửi bởi minhtuyb on 11-03-2012 - 17:50 trong Bất đẳng thức và cực trị

Bài 1: $S=\sum \frac{1}{xy}\geq \frac{9}{xy+yz+zx}\geq \frac{9}{x^2+y^2+z^2}$ (Áp dụng BĐT $a^2+b^2+c^2\geq ab+bc+ca$) $\geq \frac{9}{3}=3$
Dấu bằng xảy ra khi $x=y=z=1$
Vậy $minS=3$ khi $x=y=z=1$
Bài 2: $S=(x+z)(y+t)\leq \frac{(x+y+z+t)^2}{4}=\frac{(1.x+1.y+\frac{1}{\sqrt{2}}.z\sqrt{2}+\frac{1}{\sqrt{2}}.t\sqrt{2})^2}{4}$
$\leq ^{Cauchy-Schwarz}\frac{(1^2+1^2+\frac{1}{2}+\frac{1}{2})(x^2+y^2+2z^2+2t^2)}{4}=\frac{3}{4}$
Bài toán xong, dấu bằng vô tỉ nên lười tìm :P
Bài 3: Ghép Cauchy chăng ?



#303641 Xác định a,b,c để phương trình $ax^{2}+bx+c=0$ có nghiệm thuộc đoạn...

Đã gửi bởi minhtuyb on 11-03-2012 - 20:09 trong Bất đẳng thức và cực trị

Câu đâu: Để pt có nghiệm thì $\Delta=b^2-4ac\geq 0$
Gọi $x_1;x_2$ là 2 nghiệm của pt (không nhất thiết phân biệt), để $x_1;x_2\geq 0$ thì:
$\left\{\begin{matrix}x_1+x_2=-\frac{b}{a}\geq 0\\x_1x_2=\frac{c}{a}\geq 0\end{matrix}\right.$
-Để $x_1;x_2\leq 1$ thì:
$\left\{\begin{matrix}(1-x_1)+(1-x_2)\geq 0\\(1-x_1)(1-x_2)\geq 0\end{matrix}\right.$
Kết hợp lại ta sẽ có điều kiện chung của $a,b,c$ :D



#303644 Chứng minh rằng $B = 3 + {3^3} + {3^5} + ... + {3^{1991}} \vdots 13...

Đã gửi bởi minhtuyb on 11-03-2012 - 20:26 trong Đại số

$B=(3+3^3+3^5)+(3^7+3^9+3^{11})+...+(3^{1987}+3^{1989}+3^{1991})$
$=3(1+3^2+3^4)+3^7(1+3^2+3^4)+...+3^{1987}(1+3^2+3^4)$
$=91(3+3^7+...+3^{1987})\vdots 13$ (Do $91\vdots 13$)



#303733 Topic bất đẳng thức THCS (2)

Đã gửi bởi minhtuyb on 12-03-2012 - 11:40 trong Bất đẳng thức và cực trị

chú xem lại chỗ đỏ nhé

Đúng rồi mà :D:

$\frac{a^3}{1+b^2}=\frac{a^4}{a+ab^2}$
Mà $ab=1\Rightarrow ab^2=ab.b=b\Rightarrow \frac{a^4}{a+ab^2}=\frac{a^4}{a+b}$
Tương tự với $\frac{b^4}{b+a^2b}$ thôi
Mà ai chém bài 300 đi :D



#303894 Trận 4 - "MSS04 nguyenta98ka" VS ALL

Đã gửi bởi minhtuyb on 12-03-2012 - 22:25 trong Thi giải toán Marathon cấp THCS 2012

Huy trận này không bắt xe về kịp rồi :D
Trở lại với top 4 :)
Mà mấy bài hình như hơi quan trọng hóa vấn đề, toàn thấy Bunhi không :P
-->yeutoan: Ta nộp trước mi mà :))
P/s: Thanks anh Hân nhiều :x. May cho MSS đã có một trọng tài chính như anh :x



#303991 Chứng minh rằng $$\frac{1}{a}+\frac{1}{b}+\frac{1}{c...

Đã gửi bởi minhtuyb on 13-03-2012 - 20:10 trong Bất đẳng thức và cực trị

Bài này chắc là bạn tự chế, chứ mình thấy ý tưởng đánh giá $a-4<0$ để ghép vào$(a-4)(a-1)^2\leq 0$ không được rõ cho lắm :(



#303996 Truyện 8/3 ( Của PSW)

Đã gửi bởi minhtuyb on 13-03-2012 - 20:17 trong Quán hài hước

Like mạnh =))
Hê hê anh PSW chém gió kinh quá :D



#304031 Giải hpt: $\left\{\begin{matrix} x^{4}+y^{2}=\frac{...

Đã gửi bởi minhtuyb on 13-03-2012 - 21:33 trong Phương trình, hệ phương trình và bất phương trình

Tham khảo câu 2 ở đây: http://diendan.hocma...311&postcount=3. Đề trông không giống lắm nhưng chắc cách làm tương tự :D



#304136 Giải phương trình:$$\sqrt[4]{x-2}+\sqrt[4]{4-x}=2$...

Đã gửi bởi minhtuyb on 14-03-2012 - 12:45 trong Phương trình - hệ phương trình - bất phương trình

Theo AM-GM :D:
$VT=\sqrt[4]{1.1.1.(x-2)}+\sqrt[4]{1.1.1.(4-x)}\leq \frac{3+x-2}{4}+\frac{3+4-x}{4}=2=VP$
Dấu bằng xảy ra khi $x=3$



#304158 Tìm số tự nhiên n có 2 CS. Biết 2n+1 và 3n+1 là các số chính phương

Đã gửi bởi minhtuyb on 14-03-2012 - 15:03 trong Đại số

3/ Nếu $x^2$ là số chính phương chẵn thì được viêt dưới dạng $4n$
Nếu $x^2$ là số chính phương lẻ: $x^2=(2k+1)^2=4k^2+4k+1=4k(k+1)+1$ được viết dưới dạng $4n+1$
$\Rightarrow Q.E.D$
4/ Xét tích:
$(2^n-1).2^n.(2^n+1)\vdots 3$ (Vì là tích 3 số tự nhiên liên tiếp)
Mà $2^n\not\vdots 3;2^n+1\not\vdots 3$(Vì là số nguyên tố) $\Rightarrow 2^n-1\vdots 3\Rightarrow Q.E.D$
5/ $10\equiv 1(mod9)\to 10^{2012}\equiv 1^{2012}=1(mod9)$
$\Rightarrow 10^{2012}+53\equiv 1+53\equiv 0(mod9)\Rightarrow$ số đã cho là số tự nhiên